L’Hôpital’s Rule for indeterminate powers

In summary: However, the limit of the logarithm at 1 is not interesting, as it is the same as the limit of the original expression. The swapping occurs in going from ##\lim_{x \to 0^+} \ln y = \dots = 4## to the next equation which isn't shown. Namely, ##\lim_{x \to 0^+} \ln y = \ln(\lim_{x \to 0^+} <\text{stuff}>) = 4##. Here the limit operation and natural log are swapped.
  • #1
member 731016
Homework Statement
Please see below
Relevant Equations
Please see below
For this,
1684824210724.png

Does someone please know why we are allowed to swap the limit as x approaches zero from the right of y with that of In y?

Thank you for any help!
 
Physics news on Phys.org
  • #2
Hi,
Isnt it so that if the limit of the logarithm is 4, then the limit the exercise asks for is ##e^4## ?
I don't see any swapping
x1+sin(4x)( )^cot xe^4
0.1​
1.389418​
26.51954​
54.59815​
0.01​
1.039989​
50.44658​
0.001​
1.004​
54.16361​
0.0001​
1.0004​
54.55449​
0.00001​
1.00004​
54.59378​

##\ ##
 
Last edited:
  • Like
Likes FactChecker and member 731016
  • #3
Continuity of the function ln() at 1. Of course, the answer will have to be converted back, so 4 is not the final answer.
 
  • Like
Likes member 731016
  • #4
I think it's continuity of exp we need, since we're not pulling the limit inside the log. [tex]
\lim_{x \to a} f(x) = \lim_{x \to a} \exp(\ln f(x)) = \exp(\lim_{x \to a} \ln f(x)).[/tex]
 
  • Like
Likes member 731016, SammyS and FactChecker
  • #5
pasmith said:
I think it's continuity of exp we need, since we're not pulling the limit inside the log. [tex]
\lim_{x \to a} f(x) = \lim_{x \to a} \exp(\ln f(x)) = \exp(\lim_{x \to a} \ln f(x)).[/tex]
Good point. But I think we need the continuity of the log at one point and the continuity of exp at the end to undo what was done with the log.
 
  • Like
Likes member 731016
  • #6
Clearly it doesn't help us to look at [itex]g\left(\lim_{x \to a} g^{-1}(f(x))\right)[/itex] if we can't say anything about the behaviour of [itex]g^{-1}\circ f[/itex] near [itex]a[/itex].
 
  • Like
Likes member 731016 and FactChecker
  • #7
pasmith said:
Clearly it doesn't help us to look at [itex]g\left(\lim_{x \to a} g^{-1}(f(x))\right)[/itex] if we can't say anything about the behaviour of [itex]g^{-1}\circ f[/itex] near [itex]a[/itex].
I see your point. Although the continuity of ln() is good motivation for taking logs, it is not an essential part of the proof.
 
  • Like
Likes member 731016
  • #8
BvU said:
Isnt it so that if the limit of the logarithm is 4, then the limit the exercise asks for is e4?
I don't see any swapping
The swapping occurs in going from ##\lim_{x \to 0^+} \ln y = \dots = 4## to the next equation which isn't shown. Namely, ##\lim_{x \to 0^+} \ln y = \ln(\lim_{x \to 0^+} <\text{stuff}>) = 4##. Here the limit operation and natural log are swapped.
From this we conclude that the original expression has a limit of ##e^4##.
FactChecker said:
Although the continuity of ln() is good motivation for taking logs, it is not an essential part of the proof.
Taking logs of both sides is the standard way of dealing with the limits of exponential expressions where the variable occurs in the exponent and possibly elsewhere.
 
  • Like
Likes member 731016

Related to L’Hôpital’s Rule for indeterminate powers

What is L’Hôpital’s Rule for indeterminate powers?

L’Hôpital’s Rule for indeterminate powers is a mathematical method used to evaluate limits of forms that are indeterminate, such as 0^0, ∞^0, and 1^∞. It involves taking the natural logarithm of the function, transforming the problem into a form that can be handled by L’Hôpital’s Rule for indeterminate quotients, and then exponentiating the result to find the limit.

How do you apply L’Hôpital’s Rule to an indeterminate power?

To apply L’Hôpital’s Rule to an indeterminate power, follow these steps: 1) Let the limit be L and the function be f(x)^g(x). 2) Take the natural logarithm of the function, ln(L) = ln(f(x)^g(x)) = g(x) * ln(f(x)). 3) Evaluate the limit of g(x) * ln(f(x)) using L’Hôpital’s Rule if it is in an indeterminate form like 0*∞. 4) Exponentiate the result to find the original limit, L = e^(limit found in step 3).

In which cases can L’Hôpital’s Rule for indeterminate powers not be applied?

L’Hôpital’s Rule for indeterminate powers cannot be applied if the function does not result in an indeterminate form of 0^0, ∞^0, or 1^∞. Additionally, if the transformed limit using the natural logarithm does not result in an indeterminate form suitable for L’Hôpital’s Rule (like 0/0 or ∞/∞), then the rule cannot be applied.

What are some common mistakes to avoid when using L’Hôpital’s Rule for indeterminate powers?

Common mistakes include: 1) Not correctly identifying the indeterminate form. 2) Forgetting to take the natural logarithm of the function before applying L’Hôpital’s Rule. 3) Failing to exponentiate the result after applying L’Hôpital’s Rule. 4) Misapplying L’Hôpital’s Rule to forms that are not 0/0 or ∞/∞ after taking the natural logarithm.

Can L’Hôpital’s Rule be used for limits involving trigonometric functions in indeterminate powers?

Yes, L’Hôpital’s Rule can be used for limits involving trigonometric functions in indeterminate powers. The process remains the same: take the natural logarithm of the function, simplify the expression, and then apply L’Hôpital’s Rule to the resulting limit if it is in an indeterminate form. Finally, exponentiate the result to find the original limit.

Similar threads

Replies
1
Views
699
  • Calculus and Beyond Homework Help
Replies
14
Views
1K
  • Calculus and Beyond Homework Help
Replies
23
Views
1K
  • Calculus and Beyond Homework Help
Replies
34
Views
2K
  • Calculus and Beyond Homework Help
Replies
17
Views
1K
  • Calculus and Beyond Homework Help
Replies
3
Views
2K
  • Calculus and Beyond Homework Help
Replies
5
Views
2K
  • Calculus and Beyond Homework Help
Replies
9
Views
2K
  • Calculus and Beyond Homework Help
Replies
5
Views
1K
  • Calculus and Beyond Homework Help
Replies
16
Views
2K
Back
Top